Đến nội dung

Hình ảnh

$\frac{a}{a+b+1}+\frac{b}{b+c+1}+\frac{c}{c+a+1}\leq1$

đại

  • Please log in to reply
Chủ đề này có 5 trả lời

#1
trungheosocute36

trungheosocute36

    Binh nhất

  • Thành viên
  • 28 Bài viết

$\frac{1}{a}$+$\frac{1}{b}$+$\frac{1}{c}$=a+b+c
Cm: 5(a+b+c) $\geq$7+8abc

Bài 2: Cho $a+b+c=3$ (a,b,c là các số thực dương)
Cm: $\frac{a}{a+b+1}$+$\frac{b}{b+c+1}$+$\frac{c}{c+a+1}$$\leq$1

 


Bài viết đã được chỉnh sửa nội dung bởi trungheosocute36: 04-08-2015 - 22:27


#2
Hoang Nhat Tuan

Hoang Nhat Tuan

    Hỏa Long

  • Thành viên
  • 974 Bài viết

$\frac{1}{a}$+$\frac{1}{b}$+$\frac{1}{c}$=a+b+c
Cm: 5(a+b+c) $\geq$7+8abc

Bài 2: Cho $a+b+c=3$ (a,b,c là các số thực dương)
Cm: $\frac{a}{a+b+1}$+$\frac{b}{b+c+1}$+$\frac{c}{c+a+1}$$\leq$1

Bài 1: Hình như thiếu điều kiện $a,b,c>0$

Giả thiết viết lại thành: $ab+bc+ca=abc(a+b+c)$ (1)

Từ giả thiết cũng rút ra được $a+b+c \geq 3$

Sử dụng (1) viết BĐT lại thành:

$5(a+b+c)^2\geq 7(a+b+c)+8(ab+bc=ca)$

Lại có: $(a+b+c)^2\geq 3(ab+bc+ca)$

Cần chứng minh: $5(a+b+c)^2\geq 7(a+b+c)+\frac{8(a+b+c)^2}{3}<=>a+b+c\geq 3$ (đã được chứng minh ở trên)

BĐT được chứng minh


Ngài có thể trói cơ thể tôi, buộc tay tôi, điều khiển hành động của tôi: ngài mạnh nhất, và xã hội cho ngài thêm quyền lực; nhưng với ý chí của tôi, thưa ngài, ngài không thể làm gì được.

#3
Hoang Nhat Tuan

Hoang Nhat Tuan

    Hỏa Long

  • Thành viên
  • 974 Bài viết

$\frac{1}{a}$+$\frac{1}{b}$+$\frac{1}{c}$=a+b+c
Cm: 5(a+b+c) $\geq$7+8abc

Bài 2: Cho $a+b+c=3$ (a,b,c là các số thực dương)
Cm: $\frac{a}{a+b+1}$+$\frac{b}{b+c+1}$+$\frac{c}{c+a+1}$$\leq$1

Bài 2: Sau khi quy đồng ta thu được BĐT tương đương:

$a+b+c+1\geq a^2b+b^2c+c^2a+abc<=>a^2b+b^2c+c^2a+abc\leq 4$ (vì $a+b+c=3$ theo giả thiết)

Giả sử b nằm giữa a và c thì dễ dàng chứng minh được:

$a^2b+b^2c+c^2a\leq b(a+c)^2=\frac{1}{2}.2b.(a+c).(a+c)\leq \frac{1}{2}.\frac{8(a+b+c)^3}{27}=4$

BĐT được chứng minh


Ngài có thể trói cơ thể tôi, buộc tay tôi, điều khiển hành động của tôi: ngài mạnh nhất, và xã hội cho ngài thêm quyền lực; nhưng với ý chí của tôi, thưa ngài, ngài không thể làm gì được.

#4
trungheosocute36

trungheosocute36

    Binh nhất

  • Thành viên
  • 28 Bài viết

Bài 1: Hình như thiếu điều kiện $a,b,c>0$

Giả thiết viết lại thành: $ab+bc+ca=abc(a+b+c)$ (1)

Từ giả thiết cũng rút ra được $a+b+c \geq 3$

Sử dụng (1) viết BĐT lại thành:

$5(a+b+c)^2\geq 7(a+b+c)+8(ab+bc=ca)$

Lại có: $(a+b+c)^2\geq 3(ab+bc+ca)$

Cần chứng minh: $5(a+b+c)^2\geq 7(a+b+c)+\frac{8(a+b+c)^2}{3}<=>a+b+c\geq 3$ (đã được chứng minh ở trên)

BĐT được chứng minh

Tại sao từ giả thuyết ta suy ra được a+b+c $\geq$ 3 vậy


Bài viết đã được chỉnh sửa nội dung bởi trungheosocute36: 05-08-2015 - 14:32


#5
Dinh Xuan Hung

Dinh Xuan Hung

    Thành viên nổi bật 2015

  • Thành viên nổi bật 2016
  • 1396 Bài viết

Tại sao từ giả thuyết ta suy ra được a+b+c $\geq$ 3 vậy

Ta có:$a+b+c=\frac{1}{a}+\frac{1}{b}+\frac{1}{c}\geq \frac{9}{a+b+c}\Leftrightarrow (a+b+c)^2\geq 9\Leftrightarrow a+b+c\geq 3(\forall a,b,c>0)$



#6
81NMT23

81NMT23

    Hạ sĩ

  • Thành viên
  • 61 Bài viết

$\frac{1}{a}$+$\frac{1}{b}$+$\frac{1}{c}$=a+b+c
Cm: 5(a+b+c) $\geq$7+8abc

Bài 2: Cho $a+b+c=3$ (a,b,c là các số thực dương)
Cm: $\frac{a}{a+b+1}$+$\frac{b}{b+c+1}$+$\frac{c}{c+a+1}$$\leq$1

Bài 2:

Lấy 3 trừ đi bất đẳng thức ban đầu ta có phép biến đổi tương đương:

$\sum \frac{b}{a+b+1}+\sum \frac{1}{a+b+1}\geqslant 2$

Sử dụng bất đẳng thức Bunhiacopski 3 số ta có:

$\Leftrightarrow \frac{(a+b+c)^{2}}{ab+bc+ca+a+b+c+a^{2}+b^{2}+c^{2}}+\frac{9}{2(a+b+c)+3}\geqslant 2$

Thay $a+b+c=3$ và $3\geqslant ab+bc+ca:$

$\Rightarrow \sum \frac{b}{a+b+1}+\sum \frac{1}{a+b+1}\geqslant \frac{(a+b+c)^{2}}{(a+b+c)^{2}}+1=2$

Suy ra điều phải chứng mịnh.

Dấu bằng xảy ra khi $a=b=c=1$


Bài viết đã được chỉnh sửa nội dung bởi votruc: 05-08-2015 - 20:41





0 người đang xem chủ đề

0 thành viên, 0 khách, 0 thành viên ẩn danh